Đến nội dung

Quoc Tuan Qbdh nội dung

Có 974 mục bởi Quoc Tuan Qbdh (Tìm giới hạn từ 30-03-2020)



Sắp theo                Sắp xếp  

#676257 $\sum \frac{1}{3-ab} \leq \frac...

Đã gửi bởi Quoc Tuan Qbdh on 04-04-2017 - 22:50 trong Bất đẳng thức và cực trị

Cho a,b,c là các số thực không âm thỏa mãn $a^{2}+ b^{2}+c^{2}=3$. Chứng minh rằng: 

$\sum \frac{1}{3-ab} \leq \frac{3}{2}$

Bất đẳng thức cần chứng minh tương đương với:

$$\sum \left( \dfrac{1}{3-ab} - \dfrac{1}{3} \right) \leq \dfrac{1}{2}$$ 

$$\Leftrightarrow \sum \dfrac{ab}{3\left(a^{2}+b^{2}+c^{2}-ab\right)} \leq \dfrac{1}{2}$$

Ta có hai bất đẳng thức quen thuộc sau:
$$2ab \leq a^{2}+b^{2}$$

$$4ab \leq (a+b)^{2}$$

Do đó,

$$\sum \dfrac{ab}{3\left(a^{2}+b^{2}+c^{2}-ab\right)} \leq \sum \dfrac{(a+b)^{2}}{6(a^{2}+c^{2}+b^{2}+c^{2})}$$

Mặt khác, áp dụng bất đẳng thức Cauchy-Schwarz dạng Engel ta có:
$$\sum \dfrac{(a+b)^{2}}{6(a^{2}+c^{2}+b^{2}+c^{2})} \leq \dfrac{1}{6}\sum \left( \dfrac{a^{2}}{a^{2}+c^{2}} + \dfrac{b^{2}}{b^{2}+c^{2}} \right)= \dfrac{1}{2}$$

Bất đẳng thức được chứng minh. Đẳng thức xảy ra khi $a=b=c=1$. $\blacksquare$




#661678 Tính $S_{1}=\sum_{k=0}^{1008}C^{...

Đã gửi bởi Quoc Tuan Qbdh on 12-11-2016 - 21:53 trong Mệnh đề - tập hợp

Tình các tổng sau

a) $S_{1}=\sum_{k=0}^{1008}C^{2k}_{2017}$

b) $S_{2}=\sum_{k=0}^{504}C^{4k}_{2017}$

c) $S_{3}=\sum_{k=0}^{2017}(C^{k}_{2017})^2$

d) $S_{4}=\sum_{k=0}^{2016}k^2.C^{k}_{2016}$

a) Ta có:

$$(1+1)^{2017}=C^{0}_{2017}+C^{1}_{2017}+C^{2}_{2017}+C^{3}_{2017}+C^{4}_{2017}+....+C^{2017}_{2017}$$

$$(1-1)^{2017}=C^{0}_{2017}-C^{1}_{2017}+C^{2}_{2017}-C^{3}_{2017}+C^{4}_{2017}+....-C^{2017}_{2017}$$

Suy ra, $S_1=\dfrac{2^{2017}}{2}=2^{2016}$

 

b) Ta có:

$$(1+1)^{2017}=C^{0}_{2017}+C^{1}_{2017}+C^{2}_{2017}+C^{3}_{2017}+C^{4}_{2017}+....+C^{2017}_{2017}$$

$$(1-1)^{2017}=C^{0}_{2017}-C^{1}_{2017}+C^{2}_{2017}-C^{3}_{2017}+C^{4}_{2017}+....-C^{2017}_{2017}$$

$$(1+i)^{2017}=C^{0}_{2017}+C^{1}_{2017}.i-C^{2}_{2017}-C^{3}_{2017}.i+C^{4}_{2017}+....+C^{2017}_{2017}.i$$

$$(1-i)^{2017}=C^{0}_{2017}-C^{1}_{2017}.i-C^{2}_{2017}+C^{3}_{2017}.i+C^{4}_{2017}+....-C^{2017}_{2017}.i$$

Suy ra, $S_2=\dfrac{2^{2017}+(1+i)^{2017}+(1-i)^{2017}}{4}$

Mặt khác, theo hệ thức Moivre:
$$(1+i)^{2017}=(\sqrt{2})^{2017}\left(cos\dfrac{\pi}{4}+i.sin\dfrac{\pi}{4}\right)^{2017}$$

$$=(\sqrt{2})^{2017}.\left(cos\dfrac{2017.\pi}{4}+i.sin\dfrac{2017.\pi}{4}\right)=2^{1008}(1+i)$$

$$(1-i)^{2017}=(\sqrt{2})^{2017}\left(cos\dfrac{-\pi}{4}+i.sin\dfrac{-\pi}{4}\right)^{2017}$$

$$=(\sqrt{2})^{2017}.\left(cos\dfrac{-2017.\pi}{4}+i.sin\dfrac{-2017.\pi}{4}\right)=2^{1008}(1-i)$$

Do đó, $S_2=2^{2015}+2^{1007}$.

 

c) Ta có:
$$(1+x)^{2017}.(1+x)^{2017}=(C^{0}_{2017}+C^{1}_{2017}.x+....+C^{2016}_{2017}.x^{2016}+C^{2017}_{2017})(C^{2017}_{2017}+C^{2016}_{2017}.x^{2016}+....+C^{1}_{2017}.x+C^{0}_{2017})$$

$$(1+x)^{4034}=C^{0}_{4034}+....+C^{2017}_{4034}.x^{2017}+....+C^{4034}_{4034}.x^{4034}$$

Đồng nhất hệ số của $x^{2017}$ ta được: $S_3=C^{2017}_{4034}$

 

d) Xét số hạng tổng quát:

$$k^{2}.C^{k}_{2016}=k^{2}.\dfrac{2016!}{(2016-k)!.k!}=\dfrac{2016!.(k-1+1)}{(2016-k)!.(k-1)!}=2016.2015.\dfrac{2014!}{(2016-k)!.(k-2)}+2016.\dfrac{2015!}{(2016-k)!(k-1)!}=2016.2015.C^{k-2}_{2014}+2016.C^{k-1}_{2015}$$

Đến đây thay $k=2,...,2016$ để ý $\sum_{k=0}^{n} C^{k}_{n}=(1+1)^{n}=2^{n}$.

 




#645196 $\frac{a}{ab+4}+\frac{b}{bc...

Đã gửi bởi Quoc Tuan Qbdh on 16-07-2016 - 19:49 trong Bất đẳng thức và cực trị

Cho a,b,c là các số thực dương thỏa mãn a+b+c=6.Chứng minh rằng

$\frac{a}{ab+4}+\frac{b}{bc+4}+\frac{c}{ca+4}\geq \frac{3}{4}$

Lời giải.

Bất đẳng thức cần chứng minh tương đương với:
$$\dfrac{4a}{ab+4}+\dfrac{4b}{bc+4}+\dfrac{4c}{ca+4} \geq 3$$

$$\Leftrightarrow a-\dfrac{a^{2}b}{ab+4}+b-\dfrac{b^{2}c}{bc+4}+c-\dfrac{c^{2}a}{ca+4} \geq 3$$

$$\Leftrightarrow 3 \geq \dfrac{a^{2}b}{ab+4}+\dfrac{b^{2}c}{bc+4}+\dfrac{c^{2}a}{ca+4}$$

Theo bất đẳng thức AM-GM, ta có:
$$ab+4 \geq 4\sqrt{ab} \; \; \; \; \; bc+4 \geq 4\sqrt{bc} \; \; \; \; \; ca+4 \geq 4\sqrt{ca}$$

Suy ra, $$\dfrac{1}{4} \left( \sqrt{a^{3}b}+\sqrt{b^{3}c}+\sqrt{c^{3}a} \right) \geq \dfrac{a^{2}b}{ab+4}+\dfrac{b^{2}c}{bc+4}+\dfrac{c^{2}a}{ca+4}$$

 

Bổ đề. Với các số thực dương $x;y;z$, ta luôn có bất đẳng thức sau:
$$(x^{2}+y^{2}+z^{2})^{2} \geq 3(x^{3}y+y^{3}z+z^{3}x)$$

Chứng minh. Đã có tại đây

Áp dụng Bổ đề. với $x=\sqrt{a};y=\sqrt{b};z=\sqrt{c}$, ta có:
$$3=\dfrac{1}{12}(a+b+c)^{2} \geq \dfrac{1}{4} \left( \sqrt{a^{3}b}+\sqrt{b^{3}c}+\sqrt{c^{3}a} \right)$$

$$ \geq \dfrac{a^{2}b}{ab+4}+\dfrac{b^{2}c}{bc+4}+\dfrac{c^{2}a}{ca+4}$$

Bất đẳng thức được chứng minh.

Đẳng thức xảy ra khi $a=b=c=2$.$\blacksquare$




#645191 $\frac{a^3}{b^2-bc+c^2}+\frac{b^3...

Đã gửi bởi Quoc Tuan Qbdh on 16-07-2016 - 19:31 trong Bất đẳng thức - Cực trị

Cho $a,b,c>0$. Chứng minh rằng: 

$$\frac{a^3}{b^2-bc+c^2}+\frac{b^3}{c^2-ac+a^2}+\frac{c^3}{a^2-ab+b^2} \geq \frac{3(ab+bc+ac)}{a+b+c}.$$

Lời giải đã có tại đây




#644350 $\sqrt{\frac{xy}{xy+z}} + \...

Đã gửi bởi Quoc Tuan Qbdh on 10-07-2016 - 15:14 trong Bất đẳng thức và cực trị

Cho $x,y,z> 0$ và $x+y+z=1$
chứng minh $\sqrt{\frac{xy}{xy+z}} + \sqrt{\frac{yz}{yz+x}} +\sqrt{\frac{zx}{zx+y}} \leq \frac{3}{2}$

Để ý rằng với $x+y+z=1$, ta có:
$$xy+z=xy+z(x+y+z)=xy+yz+z^{2}+zx=(z+x)(z+y)$$

Tương tự, ta cũng có:
$$yz+x=(x+y)(x+z) \; \; \; \; \; \; \; \; \; zx+y=(y+z)(y+x)$$

Do đó, theo bất đẳng thức AM-GM:

$$VT(\text{BDT})=\sqrt{\dfrac{x}{x+z}.\dfrac{y}{z+y}}+\sqrt{\dfrac{y}{x+y}.\dfrac{z}{x+z}}+\sqrt{\dfrac{z}{y+z}.\dfrac{x}{y+x}} \leq \dfrac{1}{2}\left( \dfrac{x}{x+z}+\dfrac{y}{z+y}+\dfrac{y}{x+y}+\dfrac{z}{x+z}+\dfrac{z}{y+z}+\dfrac{x}{y+x} \right)=\dfrac{3}{2}$$

Đẳng thức xảy ra khi $x=y=z=\dfrac{1}{3}$.$\blacksquare$




#636308 $((n+1)(n+2)(n+3)...(3n-1)3n) \vdots 3^{n}$ với...

Đã gửi bởi Quoc Tuan Qbdh on 28-05-2016 - 18:10 trong Số học

CMR:

$((n+1)(n+2)(n+3)...(3n-1)3n) \vdots 3^{n}$ với $n\geq 1$ (A)

Ta sẽ chứng minh bằng quy nạp.

Với $n=1$ thì (A) đúng tức là $$(n+1)...3n=2.3 \vdots 3=3^{n}$$.

Giả sử (A) đúng với $n=k, \; k \geq 2$ tức $$(k+1)(k+2)(k+3)...(3k-1)3k \vdots 3^{k}$$ Ta cần chứng minh (A) cũng đúng với $n=k+1$.

Hay: $$(k+2)(k+3)(k+4)...(3k+2)(3k+3) \vdots 3$$

Mà theo giả thiết quy nạp thì $(k+1)(k+2)(k+3)...(3k-1)3k \vdots 3^{k}$ nên ta chỉ cần chứng minh $$\dfrac{(3k+1)(3k+2)(3k+3)}{k+1} \vdots 3$$

Hay: $$(3k+1)(3k+2).3 \vdots 3$$

Hiển nhiên đúng.

Vậy theo nguyên lý quy nạp (A) được chứng minh. 




#634835 Chứng minh rằng dãy $\{u_{n}\}$ hội t...

Đã gửi bởi Quoc Tuan Qbdh on 22-05-2016 - 22:26 trong Dãy số - Giới hạn

Anh có thắc mắc là tại sao em lại nghĩ đến điều này?

À thực ra bài này ở trong đề Học sinh giỏi Tỉnh Quảng Bình lớp 11 năm 2013-2014 (vòng 2) nhưng đề Tỉnh không cho giá trị cụ thể của $u_1, u_2$.

Chỗ đó em nghĩ ghi là $u_{2n}$ và$u_{2n+1}$ cũng được.

Theo em thì từ công thức xác định dãy thì ta sẽ phải lợi dụng liên hệ giữa ba số liên tiếp trong dãy. Vì thế sẽ chia dãy $\left \{ u_n \right \}$ ra hai dãy

và lẻ tức $\left \{ u_{2n} \right \}$ và $\left \{ u_{2n+1} \right \}$.

Mặt khác, lợi dụng tính đơn điệu của hai dãy có cùng giới hạn hữu hạn: $\left \{ a_n \right \}$ và $\left \{ b_n \right \}$ ta sẽ kẹp hai số $u_{2n}$ và $u_{2n+1}$.

Nhưng em vẫn hơi thắc mắc ở chỗ xét hai dãy phụ. Anh có thể giải thích cho em được không? 




#634753 Chứng minh rằng dãy $\{u_{n}\}$ hội t...

Đã gửi bởi Quoc Tuan Qbdh on 22-05-2016 - 18:37 trong Dãy số - Giới hạn

Cho dãy $\{u_{n}\}$ xác định bởi $$\begin{cases} u_{0} = u_{1} = 1 \\ u_{n + 2} = \sqrt{u_{n + 1}} + \sqrt{u_{n}}\end{cases}$$ Chứng minh rằng dãy $\{u_{n}\}$ hội tụ và tìm giới hạn đó.

 

Ta xét hai dãy phụ:

$$(a_n):\left\{\begin{matrix}a_1=4 \\ a_{n+1}=2\sqrt{a_n} \; \;,n=1,2,3,... \end{matrix}\right.$$
$$(b_n):\left\{\begin{matrix}b_1=1 \\ b_{n+1}=2\sqrt{b_n} \; \;,n=1,2,3,... \end{matrix}\right.$$
Ta sẽ chứng minh $\lim a_n=\lim b_n=4$.
 
Thật vậy,
Ta thấy $\left \{ a_n \right \}$ là dãy hằng và $a_n=4$ nên $\lim a_n=4$.
Bây giờ, ta chỉ cần chứng minh $\lim b_n=4$.
Trước tiên, bằng quy nạp ta sẽ chứng minh được $b_n < 4$.
Với $n=1$, $b_1=1 < 4$. (đúng)
Giả sử mệnh đề đúng với $n=k \; (k \geq 2)$, ta sẽ chứng minh mệnh đề cũng đúng với $n=k+1$.
Từ công thức xác định dãy, ta có: $b_{k+1}=2\sqrt{b_k} < 2.\sqrt{4}=4$ (theo giả thiết quy nạp)
Vậy theo nguyên lý quy nạp, mệnh đề được chứng minh.
Mặt khác, từ công thức xác định dãy, ta cũng có $\left \{ b_n \right \}$ là dãy dương.
Xét hiệu, $b_{n+1}-b_{n}=\sqrt{b_n}\left( 2-\sqrt{b_n} \right) < 0$ (vì $b_n \leq 4$)
Do đó, $\left \{ b_n \right \}$ là dãy tăng, bị chặn trên bởi $4$ nên tồn tại giới hạn hữu hạn $\lim b_n =4$.
 
Tiếp theo, ta sẽ chứng minh $$b_n \leq \text{min} \left \{ u_{2n}, \; u_{2n+1} \right \} \leq \text{max} \left \{ u_{2n}, \; u_{2n+1} \right \} \leq a_n$$
Với $n=0$, ta có: $1=b_1 \leq \text{min} \left \{ u_0; \; u_1 \right \} \leq \text{max} \left \{ u_0; \; u_1 \right \} \leq a_1=4$. (đúng)
Giả sử mệnh đề đúng với $n=k \;, k=1,2,3,...$ tức là
$$b_k \leq \text{min}\left \{ u_{2k}, \; u_{2k+1} \right \} \leq \text{max}\left \{ u_{2k}, \; u_{2k+1} \right \} \leq a_k$$
Khi đó, $$u_{2k+2}=\sqrt{u_{2k}}+\sqrt{u_{2k+1}} \geq 2\sqrt{b_k} =b_{k+1}$$
$$\Rightarrow u_{2k+3}=\sqrt{u_{2k+1}}+\sqrt{u_{2k+2}} \geq \sqrt{b_k}+\sqrt{b_{k+1}} > 2 \sqrt{b_{k}}=b_{k+1}$$
Tương tự thì, 
$$u_{2k+2}=\sqrt{u_{2k}}+\sqrt{u_{2k+1}} \leq 2\sqrt{a_k} =a_{k+1}$$
$$\Rightarrow u_{2k+3}=\sqrt{u_{2k+1}}+\sqrt{u_{2k+2}} \leq \sqrt{a_k}+\sqrt{a_{k+1}} = 2 \sqrt{a_{k}}=a_{k+1}$$
Do đó, mệnh đề cũng đúng với $n=k+1$.
Vậy theo nguyên lý quy nạp, mệnh đề được chứng minh.
 
Mặt khác, theo chứng minh ở đầu bài ta có $\lim a_n=\lim b_n=4$,
Suy ra, theo nguyên lý kẹp, ta có $\lim u_{2n}=\lim u_{2n+1}=4$.
Nên dãy $\left \{ u_n \right \}$ hội tụ và tồn tại giới hạn hữu hạn $\lim u_n =4$.$\blacksquare$



#634286 Cho $a,b,c>0$.CMR $\sum \frac{1}{...

Đã gửi bởi Quoc Tuan Qbdh on 20-05-2016 - 15:46 trong Bất đẳng thức và cực trị

Cho $a,b,c>0$.Chứng minh rằng:         

$\frac{1}{a+b}+\frac{1}{b+c}+\frac{1}{c+a} \geq \frac{3}{3a+2b+c}+\frac{3}{3b+2c+a}+\frac{3}{3c+2a+b}$

Áp dụng bất đẳng thức Cauchy-Schwarz, ta có:
$$\dfrac{1}{a+b}+\dfrac{1}{a+b}+\dfrac{1}{c+a} \geq \dfrac{9}{3a+2b+c}$$

$$\dfrac{1}{b+c}+\dfrac{1}{b+c}+\dfrac{1}{a+b} \geq \dfrac{9}{3b+2c+a}$$

$$\dfrac{1}{c+a}+\dfrac{1}{c+a}+\dfrac{1}{b+c} \geq \dfrac{9}{3c+2a+b}$$

Cộng vế theo vế của các bất đẳng thức trên, chia $3$ ở cả hai vế ta được bất đẳng thức cần chứng minh.

Đẳng thức xảy ra khi $a=b=c$.$\blacksquare$

 



#631519 Chứng minh: $\sum \frac{a}{b(4c+15)(b+2c)^...

Đã gửi bởi Quoc Tuan Qbdh on 05-05-2016 - 23:07 trong Bất đẳng thức - Cực trị

Cho a;b;c là các số thực không âm thỏa mãn:

$(a+b+2c)(a+2b+c)(2a+b+c)=1$

Chứng minh:

$\sum \frac{a}{b(4c+15)(b+2c)^{2}}\geq \frac{1}{3}$

Đã được giải tại đây: http://diendantoanho...2c2-geq-frac13/




#631192 $x^2+y^2+z^2+\sqrt{xyz}( \sqrt x + \sqrt y +...

Đã gửi bởi Quoc Tuan Qbdh on 04-05-2016 - 16:09 trong Bất đẳng thức và cực trị

Cho $x,y,z$ là các số thực ko âm. Chứng minh :

$$x^2+y^2+z^2+\sqrt{xyz}( \sqrt x + \sqrt y +\sqrt z) \ge 2(xy+yz+zx)$$

Theo bất đẳng thức Schur, ta có: 

$$x(\sqrt{x}-\sqrt{y})(\sqrt{x}-\sqrt{z})+y(\sqrt{y}-\sqrt{z})(\sqrt{y}-\sqrt{x})+z(\sqrt{z}-\sqrt{x})(\sqrt{z}-\sqrt{y}) \geq 0$$

$$\Leftrightarrow x^{2}+y^{2}+z^{2}+\sqrt{xyz}(\sqrt{x}+\sqrt{y}+\sqrt{z}) \geq x\sqrt{xy}+x\sqrt{zx}+y\sqrt{xy}+y\sqrt{yz}+z\sqrt{zx}+z\sqrt{yz}$$

Mặt khác, theo bất đẳng thức AM-GM thì:

$$x\sqrt{xy}+y\sqrt{xy} \geq 2xy$$

$$y\sqrt{yz}+z\sqrt{yz} \geq 2yz$$

$$x\sqrt{zx}+z\sqrt{zx} \geq 2zx$$

Bài toán được chứng minh. 

Đẳng thức xảy ra khi $x=y=z$.




#628957 $\sqrt{\frac{\sum ab}{3}}...

Đã gửi bởi Quoc Tuan Qbdh on 22-04-2016 - 18:54 trong Bất đẳng thức và cực trị

Bài toán: Cho $a,b,c\geq 0$ .Chứng minh rằng:

 

$\sqrt{\frac{ab+bc+ca}{3}}\leq \sqrt[3]{\frac{(a+b)(b+c)(c+a)}{8}}$

Bổ đề. $$\dfrac{9}{8}(a+b)(b+c)(c+a) \geq (a+b+c)(ab+bc+ca)$$

Chứng minh. Ta có, theo bất đẳng thức AM-GM thì: $$(a+b)(b+c)(c+a)=(a+b+c)(ab+bc+ca)-abc \geq (a+b+c)(ab+bc+ca)-\dfrac{1}{9}(a+b+c)(ab+bc+ca)$$

$$=\dfrac{8}{9}(a+b)(b+c)(c+a)$$

Bổ đề được chứng minh.

Đẳng thức xảy ra khi $a=b=c$.

Áp dụng vào bài toán. 

Ta có: $$\sqrt[3]{\dfrac{(a+b)(b+c)(c+a)}{8}} \geq \sqrt[3]{\dfrac{(a+b+c)(ab+bc+ca)}{9}}$$

$$\geq \sqrt[3]{\dfrac{\sqrt{3(ab+bc+ca)^{3}}}{9}}=\sqrt{\dfrac{ab+bc+ca}{3}}$$

(Vì $(a+b+c)^{2} \geq 3(ab+bc+ca)$)

Bài toán được chứng minh.

Đẳng thức xảy ra khi $a=b=c$.$\blacksquare$




#628948 $\sum x^{2}+2xyz<2$

Đã gửi bởi Quoc Tuan Qbdh on 22-04-2016 - 17:44 trong Bất đẳng thức và cực trị

cho x,y,z la do dai 3 canh cua 1 tam giac co chu vi la 2.CMR:

                       $\sum x^{2}+2xyz<2$

Áp dụng bất đẳng thức tam giác, ta có:
$$x < y+z \Rightarrow 2x<x+y+z=2 \Rightarrow x-1<0$$

Tương tự ta cũng có:
$$y-1<0 \; \text{va} \; z-1<0$$

Do đó, $$(x-1)(y-1)(z-1)<0 \Leftrightarrow xyz-1-xy-yz-zx+x+y+z < 0$$ $$ \Leftrightarrow 2xyz -2 - 2(xy+yz+zx) + 2.2 < 0 $$ $$ \Leftrightarrow 2xyz +4- 2(xy+yz+zx) < 2 $$ $$ \Leftrightarrow 2xyz +(x+y+z)^{2}- 2(xy+yz+zx) < 2 $$ $$ \Leftrightarrow 2xyz +x^{2}+y^{2}+z^{2} < 2 $$




#625930 Tìm giá trị nhỏ nhất của biểu thức $P=\frac{1}{a^2+b...

Đã gửi bởi Quoc Tuan Qbdh on 08-04-2016 - 19:53 trong Bất đẳng thức và cực trị

Cho $a,b,c\geq 0$ thỏa mãn $ab+bc+ca=1$. Tìm giá trị nhỏ nhất của biểu thức $P=\frac{1}{a^2+b^2}+\frac{1}{b^2+c^2}+\frac{1}{c^2+a^2}+\frac{5}{2}(a+1)(b+1)(c+1)$

Bài toán 1. Cho các số thực không âm $a;b;c$, ta luôn có bất đẳng thức đúng sau:
$$\dfrac{1}{a^{2}+b^{2}}+\dfrac{1}{b^{2}+c^{2}}+\dfrac{1}{c^{2}+a^{2}} \geq \dfrac{10}{(a+b+c)^{2}}$$

Chứng minh. Không mất tính tổng quát, giả sử $c=min${$a;b;c$}. Lúc này, ta có các bất đẳng sau:
$$(a+\dfrac{c}{2})^{2}+(b+\dfrac{c}{2})^{2} \geq (a+0)^{2}+(b+0)^{2}=a^{2}+b^{2}$$
$$(a+\dfrac{c}{2})^{2} = a^{2}+ca+\dfrac{c^{2}}{4} \geq a^{2}+c^{2}$$

$$(b+\dfrac{c}{2})^{2} = b^{2}+bc+\dfrac{c^{2}}{4} \geq b^{2}+c^{2}$$

Do đó,

$$\dfrac{1}{a^{2}+b^{2}}+\dfrac{1}{b^{2}+c^{2}}+\dfrac{1}{c^{2}+a^{2}} \geq \dfrac{1}{(a+\dfrac{c}{2})^{2}+(b+\dfrac{c}{2})^{2}}+\dfrac{1}{(a+\dfrac{c}{2})^{2}}+\dfrac{1}{(b+\dfrac{c}{2})^{2}}$$

Đặt $\left\{\begin{matrix}x=a+\dfrac{c}{2}(x\geq0) \\ y=b+\dfrac{c}{2}(y\geq0) \end{matrix}\right.$. Lúc này, $x+y=a+b+c$.

Bây giờ ta cần chứng minh

$$\dfrac{1}{x^{2}+y^{2}}+\dfrac{1}{x^{2}}+\dfrac{1}{y^{2}} \geq \dfrac{10}{(x+y)^{2}}$$

Thật vậy,

Kết hợp sử dụng hai bất đẳng thức $AM-GM$ và $Cauchy-Schwarz$, ta có:
$$\dfrac{1}{x^{2}+y^{2}}+\dfrac{1}{x^{2}}+\dfrac{1}{y^{2}}=\dfrac{1}{x^{2}+y^{2}}+\dfrac{3}{8}\left(\dfrac{1}{x^{2}}+\dfrac{1}{y^{2}}\right)+ \dfrac{5}{8}\left(\dfrac{1}{x^{2}}+\dfrac{1}{y^{2}}\right) \geq \dfrac{1}{x^{2}+y^{2}}+\dfrac{3}{8}.\dfrac{4}{x^{2}+y^{2}}+\dfrac{5}{8}.\dfrac{2}{xy}=\dfrac{5}{2}(\dfrac{1}{x^{2}+y^{2}}+\dfrac{1}{2xy})\geq \dfrac{5}{2}.\dfrac{4}{(x+y)^{2}}=\dfrac{10}{(x+y)^{2}}$$.

Bài toán được chứng minh.

Đẳng thức xảy ra khi và chỉ khi một trong ba số $a;b;c$ bằng $0$ và hai số còn lại bằng nhau khác $0$.

 

Bài toán 2. Cho các số thực không âm $a;b;c$ thỏa mãn $ab+bc+ca=1$. Tìm giá trị nhỏ nhất của biểu thức:
$$A=(a+1)(b+1)(c+1)$$
Lời giải. Ta có:

$$A=abc+ab+bc+ca+a+b+c+1 \geq 0+1+a+b+c+1=a+b+c+2$$

 

Áp dụng vào bài toán. Theo bất đẳng thức $AM-GM$, ta có:
$$P \geq \dfrac{10}{(a+b+c)^{2}}+\dfrac{5}{2}.(a+b+c+2)=\dfrac{10}{(a+b+c)^{2}}+\dfrac{5}{4}(a+b+c)+\dfrac{5}{4}(a+b+c)+5 \geq 3.\sqrt[3]{\dfrac{10.5.5}{4.4}}+5=\dfrac{15}{2}+5=\dfrac{25}{2}$$

Vậy giá trị nhỏ nhất của biểu thức $P$ là $\dfrac{25}{2}$ khi và chỉ khi $(a;b;c)=(0;1;1);(1;0;1);(1;1;0)$.




#623207 Chứng minh rằng $\sum \frac{ab}{c+1}\leq \frac{1}{4}...

Đã gửi bởi Quoc Tuan Qbdh on 28-03-2016 - 19:36 trong Số học

Cho các số $a,b,c$ biết $a+b+c=1$. Chứng minh rằng $\frac{ab}{c+1}+\frac{bc}{a+1}+\frac{ca}{b+1}\leq \frac{1}{4}$

Ta có:

$$\frac{ab}{c+1}=\frac{ab}{a+b+c+c} \leq \frac{ab}{4}\left(\frac{1}{a+c}+\frac{1}{b+c}\right)(Cauchy-Schwarz)$$

Tương tự thì:

$$\frac{bc}{a+1} \leq \frac{bc}{4}\left(\frac{1}{a+b}+\frac{1}{c+a}\right)$$

$$\frac{ca}{b+1} \leq \frac{ca}{4}\left(\frac{1}{a+b}+\frac{1}{b+c}\right)$$

Cộng vế theo vế của các bất đẳng thức trên, ta được:
$$\frac{ab}{c+1}+\frac{bc}{a+1}+\frac{ca}{b+1} \leq \frac{1}{4}\left (\frac{bc+ca}{a+b}+\frac{ca+ab}{b+c}+\frac{ab+bc}{c+a}\right)=\frac{1}{4}$$

Đẳng thức xảy ra khi và chỉ khi $a=b=c=\frac{1}{3}$




#621736 $\dfrac{a-b}{ab+4b+}+\dfrac{b-c}...

Đã gửi bởi Quoc Tuan Qbdh on 21-03-2016 - 21:34 trong Bất đẳng thức - Cực trị

Cho $a,b,c$ là các số thực không âm. Chứng minh rằng \[\dfrac{a-b}{ab+4b+4}+\dfrac{b-c}{bc+4c+4}+\dfrac{c-a}{ca+4a+4}\geq 0\] 

 Ta có bất đẳng thức cần chứng minh tương đương:
$$\frac{ab+4a+4}{ab+4b+4}+\frac{bc+4b+4}{bc+4c+4}+\frac{ca+4c+4}{ca+4a+4} \geq 3$$

$$\Leftrightarrow \frac{ab+2a+2b+4}{ab+4b+4}+\frac{bc+2b+2c+4}{bc+4b+4}+\frac{ca+2c+2a+4}{ca+4a+4} \geq 3$$

$$\Leftrightarrow \frac{(a+2)(b+2)}{ab+4b+4}+\frac{(b+2)(c+2)}{bc+4b+4}+\frac{(c+2)(a+2)}{ca+4a+4} \geq 3(*)$$

 Ta có đẳng thức sau:

$$3(a+2)(b+2)(c+2)=(ab+4b+4)(c+2)+(bc+4c+4)(a+2)+(ca+4a+4)(b+2)$$

 Do đó, theo BĐT $Cauchy-Schwarz$ thì:
$$VT(*)=(a+2)(b+2)(c+2)\left ( \frac{1}{(c+2)(ab+4b+4)}+\frac{1}{(a+2)(bc+4b+4)}+\frac{1}{(b+2)(ca+4a+4)} \right ) $$

$$\geq (a+2)(b+2)(c+a)\frac{9}{(ab+4b+4)(c+2)+(bc+4c+4)(a+2)+(ca+4a+4)(b+2)}=3=VP(*)$$

 Bài toán được chứng minh. Đẳng thức xảy ra khi và chỉ khi $a=b=c$.




#621672 Hỏi có thể sắp xếp được hay không tất cả các chữ số này thành một dãy sao cho...

Đã gửi bởi Quoc Tuan Qbdh on 21-03-2016 - 19:22 trong Số học

ĐỀ THI TUYỂN SINH VÀO LỚP 10, TRƯỜNG PHỔ THÔNG NĂNG KHIẾU ĐHQG TPHCM, 2005-2006.

Dưới đây là câu cuối cùng của đề:

 Xét 81 chữ số, trong đó có 9 chữ số 1    ;   9 chữ số 2;...  ;  9 chữ số 9. Hỏi có thể sắp xếp được hay không tất cả các chữ số này thành một dãy sao cho với mọi $k=1,2,...,9$ trong mỗi khoảng giữa hai chữ số $k$ liên tiếp ở trên dãy có đúng $k$ chữ số ?

 Giả sử có thể sắp xếp được $81$ chữ số đã cho thành một dãy sao cho với mọi $k=1,2,..,9$ trong mỗi khoảng giữa hai chữ số $k$ liên tiếp ở trên dãy có đúng $k$ chữ số.

 Xét với $k=9$ ta có tất cả $8$ khoảng giữa các chữ số $9$ liên tiếp ở trên dãy và mỗi khoảng có đúng $9$ chữ số.

Như vậy, giữa hai chữ số $9$ ở đầu bên phải và đầu bên trái của dãy bao gồm $8$ khoảng, mỗi có $9$ chữ số khác $9$ và $7$ chữ số $9$, tổng cộng là: $8.9+7=79$(số)

Như thế, hai chữ số $9$ ở đầu bên phải và đầu bên trái dãy nằm ở các vị trí đầu và cuối của dãy. 

 Xét với $k=8$ ta có dãy được tạo bởi hai số và các chữ số ở giữa chúng có tổng cộng: $2+8=10$(số)

Do đó, trong một khoảng giữa hai chữ số $9$ liên tiếp không thể có đồng thời hai chữ số $8$.

Mặt khác, trong dãy chỉ có $8$ khoảng giữa các chữ số $9$ liên tiếp nên chỉ có thể có $8$ chữ số $8$ ứng với $8$ khoảng đó.$(KTM)$

 Vậy không thế sắp xếp được $81$ chữ số đã cho thành một dãy sao cho với mọi $k=1,2,...,9$ trong mỗi khoảng giữa hai chữ số $k$ liên tiếp ở trên dãy có đúng $k$ chữ số. 

 

Bằng cách giải tương tự cũng có thể giải quyết bài toán sau:

Xét $n^{2}$ chữ số, trong đó có $n$ chữ số $1$; $n$ chữ số $2$;...; $n$ chữ số $n$. Hỏi có thể sắp xếp được hay không tất cả các chữ số này thành một dãy sao cho với mọi $k=1,2,...,n$ trong mỗi khoảng giữa hai chữ số $k$ liên tiếp ở trên dãy có đúng $k$ chữ số ? ( với $n$ là một số tự nhiên khác $0$ cho trước )




#621517 $\lim_{x \to 1}\left ( \frac{m}...

Đã gửi bởi Quoc Tuan Qbdh on 20-03-2016 - 21:33 trong Dãy số - Giới hạn

Cho $m;n$ là hai số nguyên dương khác nhau. Tính

 

$\lim_{x \to 1}\left ( \dfrac{m}{1-x^m}-\dfrac{n}{1-x^n} \right )$

Ta có:
$\lim_{x \to 1} (\frac{m}{1-x^{m}}-\frac{n}{1-x^{n}}) = \lim_{x \to 1} (\frac{m}{1-x^{m}}-\frac{1}{1-x}-\frac{n}{1-x^{n}}+\frac{1}{1-x})$
$= \lim_{x \to 1} (\frac{1-x^{m-1}+1-x^{m-2}+...+1-x+1-1}{1-x^{m}}-\frac{1-x^{n-1}+1-x^{n-2}+...+1-x+1-1}{1-x^{n}})$

$= \lim_{x \to 1} (\frac{1+x+...+x^{m-2}+1+x+..+x^{m-3}+...+1}{1+x+...+x^{m-1}}-\frac{1+x+...+x^{n-2}+1+x+..+x^{n-3}+...+1}{1+x+...+x^{n-1}})$

$=\frac{m-1+m-2+...+1}{m}-\frac{n-1+n-2+...+1}{n}=\frac{m(m-1)}{2m}-\frac{n(n-1)}{2n}=\frac{m-n}{2}$




#621481 CM: $\sum \sqrt{4-a^{2}} \leq 3\sqrt{3}$.

Đã gửi bởi Quoc Tuan Qbdh on 20-03-2016 - 20:51 trong Bất đẳng thức và cực trị

 

2. Cho a,b,c $\in \left [ -2;2 \right ]$  thỏa mãn: $a+b+c=3$. CM: $\sum \sqrt{4-a^{2}} \leq 3\sqrt{3}$.

 

Áp dụng BĐT $Cauchy-Schwarz$ ta có:
$\sqrt{4-a^{2}}+\sqrt{4-b^{2}}+\sqrt{4-c^{2}}=\sqrt{2-a}.\sqrt{2+a}+\sqrt{2-b}.\sqrt{2+b}+\sqrt{2-c}.\sqrt{2+c} \leq \sqrt{(2-a+2-b+2-c)(2+a+2+b+2+c)}=3\sqrt{3}$
Đẳng thức xảy ra khi và chỉ khi $a=b=c=1$




#621469 Topic về phương trình và hệ phương trình

Đã gửi bởi Quoc Tuan Qbdh on 20-03-2016 - 20:38 trong Phương trình - hệ phương trình - bất phương trình

Bài 341: $x^{4}+x^{2}+(x^{2}+2x-1)^{3}=2-4x+2\sqrt[3]{x^{2}-x^{4}}$

 Phương trình đã cho tương đương:

$(x^{2}-x^{4})+2\sqrt[3]{x^{2}-x^{4}}=(x^{2}+2x-1)^{3}+2(x^{2}+2x-1)$
 Xét: $f(t)=t+2\sqrt[3]{t},\forall t \in \mathbb{R}$

Ta có: $f'(t)=1+\frac{2}{3\sqrt[3]{t^{2}}}>0, \forall t \in \mathbb{R}$

Nên $f(t)$ đồng biến trên $\mathbb{R}$

 Do đó, $f(x^{2}-x^{4})=f((x^{2}+2x-1)^{3})$
$\Leftrightarrow x^{2}-x^{4}=(x^{2}+2x-1)^{3}$

$\Leftrightarrow x^{2}-x^{4}=x^{6}+6x^{5}+9x^{4}-4x^{3}-9x^{2}+6x-1$
$\Leftrightarrow x^{6}+6x^{5}+10x^{4}-4x^{3}-10x^{2}+6x-1=0$
 Nhận thấy $x=0$ không là nghiệm của phương trình.

 Với $x$ khác $0$, chia cả hai vế của hệ cho $x^{3}$ ta được:
$x^{3}+6x^{2}+10x-4-\frac{10}{x}+\frac{6}{x^{2}}-\frac{1}{x^{3}}=0$

$\Leftrightarrow (x^{3}-3x+\frac{3}{x}-\frac{1}{x^{3}})+6(x^{2}+\frac{1}{x^{2}}-2)+13(x-\frac{1}{x})+8=0$
 Đặt $a=x-\frac{1}{x}$, phương trình trở thành:
$a^{3}+6a^{2}+13a+8=0$
$\Leftrightarrow (a+1)(a^{2}+5a+8)=0$
$\Leftrightarrow a=-1$(Vì $a^{2}+5a+8=(a+\frac{5}{2})^{2}+\frac{7}{4}>0$).

 Suy ra: $x-\frac{1}{x}=-1 \Leftrightarrow x^{2}+x-1=0 \Leftrightarrow x=\frac{-1-\sqrt{5}}{2}$ hoặc $x=\frac{-1+\sqrt{5}}{2}$

 Vậy tập nghiệm của phương trình là $S=${$\frac{-1-\sqrt{5}}{2};\frac{-1+\sqrt{5}}{2}$}.
 




#620624 $(a+\frac{1}{b})(b+\frac{1}...

Đã gửi bởi Quoc Tuan Qbdh on 16-03-2016 - 20:42 trong Bất đẳng thức và cực trị

Cho a,b,c>0, a+b+c=1. CMR: $(a+\frac{1}{b})(b+\frac{1}{c})(c+\frac{1}{a}) \geq (\frac{10}{3})^3$

Áp dụng BĐT $AM-GM$ ta có:

$a+9.\frac{1}{9b} \geq 10\sqrt[10]{\frac{a}{9^{9}.b^{9}}}$

$b+9.\frac{1}{9c} \geq 10\sqrt[10]{\frac{b}{9^{9}.c^{9}}}$

$c+9.\frac{1}{9a} \geq 10\sqrt[10]{\frac{c}{9^{9}.a^{9}}}$

Suy ra: $(a+\frac{1}{b})(b+\frac{1}{c})(c+\frac{1}{a}) \geq 10^{3}\sqrt[10]{\frac{1}{9^{27}a^{8}b^{8}c^{8}}}$

Lại có: $a^{8}b^{8}c^{8} \leq (\frac{a+b+c}{3})^{24}=\frac{1}{9^{12}}$ 

Do đó, $(a+\frac{1}{b})(b+\frac{1}{c})(c+\frac{1}{a}) \geq 10^{3}\sqrt[10]{\frac{1}{9^{15}}}=(\frac{10}{3})^{3}$
Đẳng thức xảy ra khi $a=b=c=\frac{1}{3}$

 

 

Cũng có thể giải bằng BĐT $Holder$
Theo BĐT $Holder$ ta có:
$(a+\frac{1}{b})(b+\frac{1}{c})(c+\frac{1}{a}) \geq (\sqrt[3]{abc}+\frac{1}{\sqrt[3]{abc}})^{3}$

Áp dụng BĐT $AM-GM$ lại có:
$\sqrt[3]{abc}+\frac{1}{9\sqrt[3]{abc}} \geq \frac{2}{3}$

$\frac{8}{9\sqrt[3]{abc}} \geq \frac{8}{3(a+b+c)}=\frac{8}{3}$
Từ đó suy ra điều phải chứng minh.

Dấu bằng xảy ra khi và chỉ khi $a=b=c=\frac{1}{3}$  




#620610 $\sum \frac{a^3}{b^2-bc+c^2}\geq...

Đã gửi bởi Quoc Tuan Qbdh on 16-03-2016 - 20:03 trong Bất đẳng thức và cực trị

Cho a,b,c >0. CMR $\frac{a^{3}}{b^{2}-bc+c^{2}}+\frac{b^{3}}{c^{2}-ac+a^{2}}+\frac{c^{3}}{a^{2}-ab+b^{2}}\geq \frac{3\left ( ab+bc+ca \right )}{a+b+c}$

Ta có: $VT=\frac{a^{4}}{ab^{2}-abc+ac^{2}}+\frac{b^{4}}{bc^{2}-abc+ba^{2}}+\frac{c^{4}}{cb^{2}-abc+ca^{2}}\geq \frac{(a^{2}+b^{2}+c^{2})^{2}}{ab^{2}+ac^{2}+bc^{2}+ba^{2}+ca^{2}+cb^{2}-3abc}(Cauchy-Schwarz)$

Dễ dàng chứng minh được $(a+b+c)^{2} \geq 3ab+3bc+3ca$ 

$\Leftrightarrow a+b+c \geq \frac{3(ab+bc+ca)}{a+b+c}$

Bây giờ cần chứng minh 

$\frac{(a^{2}+b^{2}+c^{2})^{2}}{ab^{2}+ac^{2}+bc^{2}+ba^{2}+ca^{2}+cb^{2}-3abc} \geq a+b+c$

$<=>(a^{2}+b^{2}+c^{2})^{2} \geq (ab^{2}+ac^{2}+bc^{2}+ba^{2}+ca^{2}+cb^{2}-3abc)(a+b+c)$

$<=>a^{4}+b^{4}+c^{4}+abc(a+b+c) \geq a^{3}(b+c)+b^{3}(c+a)+c^{3}(a+b)$

$<=>a^{2}(a-b)(a-c)+b^{2}(b-c)(b-a)+c^{2}(c-a)(c-b) \geq 0$ ( đúng theo BĐT $Schur$ )

Dấu bằng xảy ra khi $a=b=c$




#620179 $P = a^2 + b^2 + c^2+\frac{ab+bc+ca}{a^{2}...

Đã gửi bởi Quoc Tuan Qbdh on 14-03-2016 - 10:46 trong Bất đẳng thức và cực trị

Bài 1: Cho a, b, c là các số thực dương thay đổi thỏa mãn a+b+c=3. Tìm giá trị nhỏ nhất của $P = a^2 + b^2 + c^2+ \frac{ab+bc+ca}{a^{2}b+b^{2}c+c^{2}a}$

 

 Chú ý là từ điều kiện ta có $3(a^2+b^2+c^2)=(a+b+c)(a^2+b^2+c^2)=a^3+b^3+c^3+a^2b+b^2c+c^2a+ab^2+bc^2+ca^2$

 Áp dụng AM-GM thì $a^3+ab^2\geq 2a^2b,\ b^3+bc^2\geq 2b^2c,\ c^3+ca^2\geq 2c^2a$

 Từ đó suy ra $3(a^2+b^2+c^2)\geq 3(a^2b+b^2c+c^2a)\Leftrightarrow a^2+b^2+c^2\geq a^2b+b^2c+c^2a$

 Nên $P\geq a^2+b^2+c^2+\dfrac{ab+bc+ca}{a^2+b^2+c^2}$

 Đặt $t=a^2+b^2+c^2$ thì $t\geq 3$ và $ab+bc+ca=\dfrac{9-t}{2}$

 Ta sẽ chứng minh $t+\dfrac{9-t}{2t}\geq 4\Leftrightarrow \dfrac{(t-3)(2t-3)}{2t}\geq 0$ đúng

 Nên $P\geq 4$

 Vậy GTNN của $P$ là $4$ khi và chỉ khi $a=b=c=1$




#620157 Topic về phương trình và hệ phương trình

Đã gửi bởi Quoc Tuan Qbdh on 13-03-2016 - 23:09 trong Phương trình - hệ phương trình - bất phương trình

320.$\left\{\begin{matrix} a+b=\sqrt[3]{24} & \\ & (\sqrt{a}+\sqrt{b})(\frac{1}{\sqrt{a+3b}}+\frac{1}{\sqrt{3a+b}})=2 \end{matrix}\right.$

Ta có: Theo BĐT $AM-GM$ thì:
$\frac{\sqrt{a}}{\sqrt{a+3b}} \leq \frac{1}{2}(\frac{a}{a+b}+\frac{a+b}{a+3b})$

$\frac{\sqrt{b}}{\sqrt{a+3b}} \leq \frac{1}{2}(\frac{1}{2}+\frac{2b}{a+3b})$

$\frac{\sqrt{a}}{\sqrt{3a+b}} \leq \frac{1}{2}(\frac{1}{2}+\frac{2a}{3a+b})$

$\frac{\sqrt{b}}{\sqrt{3a+b}} \leq \frac{1}{2}(\frac{b}{a+b}+\frac{a+b}{3a+b})$

Cộng vế theo vế của các bất đẳng thức trên ta có:
$(\sqrt{a}+\sqrt{b})(\frac{1}{\sqrt{a+3b}}+\frac{1}{\sqrt{3a+b}}) \leq 2$

Nên hệ phương trình tương đương

$\left\{\begin{matrix}a+b=2.\sqrt[3]{3} \\ a=b \end{matrix}\right.\Leftrightarrow \left\{\begin{matrix}a=\sqrt[3]{3} \\ b=\sqrt[3]{3} \end{matrix}\right.$




#619631 Tìm: $\lim_{n\rightarrow+\propto }Y_{n...

Đã gửi bởi Quoc Tuan Qbdh on 10-03-2016 - 23:36 trong Dãy số - Giới hạn

Cho dãy số (Un) xác định bởi $\left\{\begin{matrix} U_{1}=3 & \\ U_{n+1}=\frac{U_{n}^{2015}+2U_{n}+4}{U_{n}^{2014}-U_{n}+6} & \end{matrix}\right.$

Đặt $Y_{n}=\sum_{i=1}^{n}\frac{1}{U_{i}^{2014}+4}$. Tìm: $\lim_{n\rightarrow+\propto }Y_{n}$

Ta có: $U_{n+1}-U_n=\frac{U_n^{2015}+2U_n+4}{U_n^{2014}-U_n+6}-U_n=\frac{(U_n-2)^{2}}{U_n^{2014}-U_n+6} >0,$ $\forall x \in N*$

Suy ra: $U_{n+1} > U_n$ suy ra $(U_n)$ là dãy tăng nên $U_{n+1}>U_n>...>U_1=3>0$ nên $(U_n)$ là dãy dương.

Giả sử $(U_n)$ tồn tại giới hạn hữu hạm là $\lim U_n=L$ ta có:

$L=\frac{L^{2015}+2L+4}{L^{2014}-L+6}\Leftrightarrow L^{2015}-L^{2}+6L=L^{2015}+2L+4 \Leftrightarrow L=2$ (vô lý vì $L>3$)

Do đó, $\lim U_n= +\infty$

Xét: $U_{n+1}-2=\frac{U_n^{2015}+2U_n+4}{U_n^{2014}-U_n+6}-2=\frac{U_n^{2015}-2U_n^{2014}+4U_n-8}{U_n^{2014}-U_n+6}$

$=\frac{(U_n-2)(U_n^{2014}+4)}{(U_n^{2014}+4)-(U_n-2)}$

$\Leftrightarrow \frac{1}{U_n-2}-\frac{1}{U_{n+1}-2}=\frac{1}{U_n^{2014}+4}$

Thay $n=1,2,...,n-1$ ta có:

$\frac{1}{U_1-2}-\frac{1}{U_2-2}=\frac{1}{U_1^{2014}+4}$

$\frac{1}{U_2-2}-\frac{1}{U_3-2}=\frac{1}{U_2^{2014}+4}$
   $. . . . .$

$\frac{1}{U_{n-1}-2}-\frac{1}{U_n-2}=\frac{1}{U_{n-1}^{2014}+4}$

Suy ra: $Y_n=\sum_{i=1}^{n} \frac{1}{U_i^{2014}+4}=\frac{1}{U_1-2}-\frac{1}{U_{n+1}-2}=1-\frac{1}{U_{n+1}-2}$
Mặt khác: $\lim U_{n+1}=+\infty$ nên $\lim \frac{1}{U_{n+1}-2}=0$

Do đó, $\lim Y_n=1$